PT50.S2.Q22 - Economist: Real wages in this country will increase

LizadvocateLizadvocate Alum Member
edited March 2019 in Logical Reasoning 29 karma

Can anyone explain Q 22 for me? Why is the right answer D & not E? I’m also confused about diagramming this one.

Thank you.

Admin note: edited title
https://7sage.com/lsat_explanations/lsat-50-section-2-question-22/

Comments

  • FixedDiceFixedDice Member
    edited March 2019 1804 karma

    1st sentence: $ increase ---> P increase -- which can be rewritten as ~(P increase) ---> ~($ increase)
    2nd sentence, 1st part (conclusion): ~($ increase)
    2nd sentence, 2nd part: Little invest

    This is a sufficient assumption question. We are looking for an answer choice that would lead us to ~($ increase). With the two premises, I would think that the correct answer choice would be something like

    Little invest ---> ~(P increase)

    and that is what (D) expresses.

    (E) is wrong because no premise discusses contributing to increasing productivity.

  • LizadvocateLizadvocate Alum Member
    29 karma

    @FixedDice thank you so much!

  • BlindReviewerBlindReviewer Alum Member
    855 karma

    Just to add on, I don't think fixating on "contributing" will lead to a fuller understanding of why E is wrong. E is wrong because it isn't strong enough to be a sufficient assumption and make the argument 100% valid -- it feels almost like a necessary assumption, but it isn't even that.

    The two premises are 1) The country is currently investing very little into new tech and 2) This pattern is going to continue in subsequent years

    What we need to do is connect these two premises to show that they do not lead to a "notable increase in productivity"

    E doesn't address the second premise, so that's the red flag. E does match up with premise 1, in that the current technology being invested in is not leading to a lot of productivity, so we're safe there, but we know nothing about what will happen in the future, given in premise 2. Just because this pattern is bound to continue doesn't mean that we won't have notable productivity (unless we say so with something like D) -- you could invest very little in tech and somehow hit a goldmine innovation that leads to productivity.

    D precludes that possibility, so it's much better than E. The reason I say D feels like a necessary assumption but it actually isn't is because of what @FixedDice points out with "contributing." I think it's a bit too strong to be a NA -- a NA would more like "current investment in technology is not increasing productivity."

  • LizadvocateLizadvocate Alum Member
    29 karma

    @BlindReviewer thank you so much for detailing that further-appreciate it!

Sign In or Register to comment.